Final Review Quiz #4

Ace your homework & exams now with Quizwiz!

Diane is a great photographer. She wants to donate one of her photos to the local art museum. Her cost for film developing, printing, mounting, and framing is $250. The art museum curator feels the photo is worth $3,000. If her AGI is $100,000, how much can she deduct for her gift to the museum? A) $0 - It is personal property. B) $250 C) $1,500 D) $3,000 E) 50% of AGI

B) $250 The tax deduction for a work of art created by the taxpayer is limited to basis. Please review the income tax deductions for gifts to charity created by the taxpayer.

Your client, Martha, requested a meeting with you. Martha is married to Glen. Glen is a workaholic. You only met him once to set up their joint account. Martha always makes all the investment decisions. Glen makes alot of money, but Martha saves a majority of it through frugal living. As the meeting starts, Martha is quite blunt. Glen has taken up with one of the girls at the plant. Cash flow to Martha from Glen has ceased. He set up a separate bank account in his name. She wants your advice on how to handle the assets in the investment account. A) Tell her to break the joint tenancy account and retitle as tenants in common. B) Tell her you need to secure statements from both her and Glen to proceed with any changes. C) Tell her to see a divorce attorney before you can proceed to change the account into her name. D) Call up Glen and ask him to come into your office.

B) Tell her you need to secure statements from both her and Glen to proceed with any changes. The account is in joint names, both are clients. She never says she wants to divorce him. Also to do Answer C, she has to get a divorce before you can proceed with the acccount titling. Do you have enough of a relationship with Glen to select Answer D? Doubtful. Answer A depends on Answer B.

Mr. Lukes owns a convenience store. After a storm hit his city, he donated $1,000 worth of food to the American Red Cross. He feels he could have sold the food for $2,000. Normally, the expenses associated with selling the food consume 50% of his profit. What amount of charitable deduction can he take? A) No contribution deduction is allowed. B) 50% of his AGI C) $1,000 D) $2,000

C) $1,000 For self-employed individuals, partnerships, and Subchapter S corporations, the contribution amount for inventory must be reduced by the amount, which would have been recognized as gain if the property had been sold by the donor at its fair market value at the time of its contribution to the charitable organization. In other words, the charitable contribution of inventory is limited to its cost.

Mr. Smart is shorting calls (at-the-money calls). What is his greatest risk? A) Time running out B) Market interest rates increasing C) The upside risk is unlimited. D) A flat market

C) The upside risk is unlimited. Naked call writing is also called shorting calls. The maximum profit is the premium income. Since the upside potential of a stock's price is unlimited, the potential loss to the writer is unlimited. A shorter defined is someone who believes that the price of the underlying stock will drop or at worse will not rise (a flat market). Mr. Smart will be able to keep the premium income. (reference Live Review Investments page 21)

In regard to the SML, beta is which of the following? A) Always a constant B) Equal to the market C) Equal to 1 D) A variable

D) A variable Beta is on the horizontal axis. It is a variable.

Daniel Munch is married with two children. He and his wife, Sonia, have the following accounts at one FDIC bank. How much FDIC insurance coverage do they have in total? Daniel: $100,000 Sonia: $100,000 Joint (Daniel and Sonia): $300,000 Daniel's IRA: $200,000 Sonia's IRA: $200,000 Daniel's Revocable Trust Child 1: $100,000 Daniel's Revocable Trust Child 2: $100,000 Sonia's Revocable Trust Child 1: $100,000 Sonia's Revocable Trust Child 2: $100,000 A) $900,000 B) $1,000,000 C) $1,100,000 D) $1,200,000 E) $1,300,000

E) $1,300,000 All the accounts are covered in full. In fact, you can have multiple beneficiaries covered as long as they are spelled out in the trust. (child 1/child 2) Retirement (IRAs) are covered up to $250,000. Separate $200,000 / Joint $300,000 / IRAs $400,000 / Trusts $400,000 = Total $1,300,000 The accounts are insured per titling, not per account.

What are the purposes of immunizing a bond portfolio? I. To reduce the bond portfolio's market losses II. To earn a specific rate of return from the bond portfolio over a given period of time, regardless of what happens to market interest rates III. To offset interest rate risk with reinvestment rate risk IV. To avoid buying zero coupon bonds A) I, II, III B) I, II, IV C) II, III, IV D) II, III E) III, IV

A) I, II, III If interest rates rise, interest rate risk causes the value of the bonds to drop, but the client can earn more on coupon payments that are reinvested. If interest rates decline, interest rate risk causes the value of the bonds to rise, but the client will earn less on coupon payments that are reinvested. The typical method of immunizing involves assembling and appropriately managing a diversified portfolio of bonds.

The CML (Capital Market Line) tells us one thing only. What does it tell us? A) The expected return on a fully diversified portfolio B) How to evaluate the performance of a single stock C) How to evaluate the performance of a portfolio that lacks full diversification D) How an individual security affects the overall risk of a portfolio

A) The expected return on a fully diversified portfolio A portfolio that is fully and effectively diversified should fall somewhere along the CML. Answers B and D are referring to the SML. The CML is used for diversified portfolios (MACRO/ uses Rp) and the SML is used for a single security.

Rod Hotshot bought a stock on a tip. His basis was $3,200. (He basis included $200 in commissions.) He sold it for $10,000 after commissions were paid. Rod is retired and is in a 12% income tax bracket. How much tax would Rod owe if he held the stock for more than one year? A) $0 B) $350 C) $1,020 D) $1,050 E) $1,500

A) $0 He is in a 12% tax bracket, the capital gains rate is zero.

Monica is age 13. She will not turn 14 until next year. She has an UTMA account set up by her grandfather. (His tax bracket is 24%.) He invests her money for her. This year the UTMA had the following income. Municipal bond income of $500 LTCG of $600 Qualified dividends of $1,200 If her parents are in a 35% tax bracket, how much is her income tax? A) $0 B) $42.50 C) $70.00 D) $134.00 E) $145.00

A) $0 Her taxable income is $1,800 ($600 plus $1,200). Her tax bracket is 10%, but the LTCGs and dividends in the 10% bracket are subject to a 0% tax. Age 14 did not matter. She is not subject to the kiddie tax as it starts at $2,300. ($1,150 free and $1,150 at 10%) Municipal bond income is tax-free income. That is why $1,800 is taxable.

Taxes are triggered for Social Security recipients when modified adjusted gross income (MAGI) exceeds specified amounts. To calculate whether Social Security benefits will be taxable, which types of income must be considered? I. Pension distributions (RMDs) II. Rental income (passive) III. Section 162 Insurance Premiums IV. Interest earned on CDs V. Tax-exempt interest from private purpose municipal bonds A) All of the above B) I, II, IV, V C) I, III, V D) II, IV, V E) III, IV

A) All of the above Rental income is income even though passive. Section 162 insurance premiums are charged to the employee> Tax-exempt (public or private) affects MAGI.

The economics of renting or buying a personal residence depend on which of the following factors? I. The prices of homes and mortgage interest rates in a particular area II. The income tax benefits of home ownership III. The extent to which home prices increase or decrease over time IV. The period of time one expects to live in the home and the degree of uncertainty of this issue A) All of the above B) I, II, IV C) II, III, IV D) I, III

A) All of the above The same type of reasoning applies to buying or leasing an auto, especially when it is a business auto.

Sid Thomas works for TTI, Inc. Sid makes $120,000 per year. TTI contracted with a disability insurance company to buy long-term disability for its key employees. The carrier agreed to insure up to 50% of salary. In Sid's case, that is up to $5,000 per month. However, TTI only agreed to pay for 60% of the $5,000 per month coverage with the provision that Sid could elect to pay for the remaining 40%. Which of the following statements is/are true? I. If Sid elects not to pay for the additional coverage, the disability benefits ($3,000) will be taxable as income. II. If Sid elects to pay for the additional coverage, 60% of the disability benefits are taxable as income, and 40% are tax-free. III. If Sid elects not to pay for the coverage, the disability benefits ($5,000) will be taxable as income. IV. If Sid elects to pay for the coverage, the disability benefits ($5,000) will be tax -free. A) I, II B) I C) II D) III E) IV

A) I, II 60% of the benefits are taxable as income (the company-paid portion of the premium), and 40% of the benefits are tax-free (if Sid pays the premium). The question says Sid could pay, not had to pay which makes Answer I correct.

Tommy Todd died this year. He was an employee of a large company.The company owned a group life policy covering him for $50,000 and the company also had a company benefit of paying a $5,000 death benefit outright. If both death benefits were paid to Tommy's wife, how much would be subject to income tax? A) $0 B) $5,000 C) $50,000 D) $55,000

B) $5,000 A group life policy is normally owned by the company. The company pays the premium, but the benefits (up to $50,000) are tax-free. The $5,000 death benefit paid the company used to be tax-free (a De Minimis fringe benefit) but is now taxable.

What is the major difference between a red herring and a prospectus? HINT: Think simple if you do not know. The question is quite easy if you eliminate the wrong answers. A) A red herring is a communist stock. B) A red herring omits the selling price and the size of the issue. C) If the front page of the prospectus is printed in red, then it is a red herring. D) The SEC does not have to approve a red herring.

B) A red herring omits the selling price and the size of the issue. Although Answer C is true, the major difference is Answer B. The SEC has to approve a red herring. Yes, questions come from material not covered.

Arthur is in a 35% income tax bracket. He has decided to buy a $40,000 car with cash. He needs to sell an investment to raise cash. Which one of the following assets would generate the least amount of tax liability if sold? NOTE: LTCGs rate is 15%. A) An annuity worth $30,000 with a basis of $28,000 B) A stock bought 6 months ago for $27,500, now worth $30,000 C) A stock bought 13 months ago for $24,000, now worth $30,000 D) A baseball card bought 2 years ago for $26,000, now worth $30,000

B) A stock bought 6 months ago for $27,500, now worth $30,000 In regards to the annuity, you cannot assume he is over age 59 ½. The question must tell you the person is over 59 1/2. You must assume it is a deferred annuity because it is worth more than he bought it for originally. It cannot be an immediate annuity. The tax is calculated as follows for each answer. A. $2,000 ordinary income at 35%, plus 10% penalty on $2,000 = $900 (under 59 1/2) B. $2,500 STCG at 35% = $875 C. $6,000 LTCG at 15% = $900 D. $4,000 at 28% = $1,120

Mr. Delay just discovered that a few years ago he made a mistake on his 1040 and is due a tax refund. What are his options? A) He can file a 1040A for the refund. B) He can file a refund claim on Form 1040X within three years from the time his original return was filed, or within two years from the time he paid the tax for that return. C) He can file an 1120 for the refund if it is within five years of the original tax filing. D) Sorry, it is too late to file a return for a refund. (One year from filing is the maximum.)

B) He can file a refund claim on Form 1040X within three years from the time his original return was filed, or within two years from the time he paid the tax for that return. There are additional rules for refunds, but Answer B is the general rule.

Mrs. Lucy, age 80, is in reasonable health. Five years ago, her husband died leaving her $3 million and placing $3.5 miillion in a bypass trust for her benefit. In addition, their home was in JTWROS. The home, FMV value $500,000 and the $4 million of investments has a high basis of $3.5 million. Mrs. Lucy has two married children and 5 grandchildren to keep her estate under $5 million. What type of asset do you recommend she give and to whom? A) Low basis, high dividend paying investments to both children and grandchildren. B) High basis, high dividend paying investments to both children and grandchildren. C) Low basis, growth investments to children and high basis, growth investments to grandchildren. D) High basis, growth investments to children and low basis, growth investments to grandchildren.

B) High basis, high dividend paying investments to both children and grandchildren. Think ages. The grandchildren have to be age 30 maybe age 40. With high basis, they can sell the investment with little or no tax or keep it and get big dividends. They could be taxed a 0% or at most 15%. Low basis investments would be subject to capital gains. If she keeps the low basis stocks until death, her estate will get a full step-up in basis. She does not have an estate tax situation.

What is a SERP? I. It is a supplemental executive retirement plan. II. It is an informally funded plan to provide benefits that greatly exceed those provided by a normal retirement plan. III. It is also called a "top hat" plan. IV. It must cover all employees for supplemental benefits. A) All of the above B) I, II, III C) I, II, IV

B) I, II, III SERPs can only be provided for management or HCEs. This is where "top hat" comes from.

Which of the following is/are considered conduit income? I. K-1 income from an S corporation II. K-1 income from a partnership III. Dividend income from a corporation IV. DNI from a trust V. K-1 loss from active participation in real estate A) I, II, III B) I, II, IV C) II, III D) II, V E) III

B) I, II, IV DNI is distributed net income. A simple trust uses the conduit principal (DNI). Answer V would have been correct if it said K-1 income. Dividends from a corporation are after-tax distributions.

While on a trip, Sandra's wedding ring disappeared. The ring was worth $10,000. She has an HO-3 policy. Which one of the following statements is true? A) The coverage on the ring is limited to $1,000. B) The coverage on the ring is limited to dollar amount ($1,000 - 1,500) if the loss is due to theft. C) If she has more than $10,000 personal property coverage, the ring is covered for its full value. D) The ring is not covered.

B) The coverage on the ring is limited to dollar amount ($1,000 - 1,500) if the loss is due to theft. The ring is only covered if its disappearance is due to theft. The dollar amount shown is not important. See Insurance prestudy lesson 3. It is a concept question (theft). There is no coverage unless a theft occurred. The theft must be reported to the police. It is a common sense question/answer. Some policies cover jewelry up to $5,000.

Mr. Golden bought a $1 million life insurance policy 10 years ago. He has paid $150,000 in premiums over 10 years. The current cash value is $175,000. The cost of the insurance over 10 years was $10,000. Mr. Golden has no use for the policy anymore. A life settlement company has offered him $400,000 in cash to buy the policy. If he sells the policy, what will be the taxable event to him? Hint: Remember the trick to life settlements. I. $25,000 taxed as ordinary income II. $225,000 taxed as ordinary income III. $235,000 taxed as long-term capital gains IV. $225,000 taxes as long-term capital gains V. $225,000 included in his estate if he dies within 3 years of the policy transfer A) I, II, V B) I, III C) I, V D) II E) IV, V

B) I, III This information is in the Live Review. Life settlements are subject to capital gains. Look for the capital gains answer that is the best way to solve this type of question. Answers III and IV are possible answers. However, answer II and V are false. Therefore, the answer has to be B. A life settlement is taxed differently than a viatical settlement. Both are the sale of the policy. The cost of insurance affects the premiums paid . $150,000 - 10,000 = $140,000 basis. Mr. Golden will have to recognize income of $260,000 (400,000 - 140,000). The income is characterized as ordinary income to the extent the cash value exceeds the premiums paid ($175,000 - 150,000) or $25,000. The balance will be treated as capital gains $260,000 - 25,000 or $235,000. HINT: With this type of question always back into the answer.

Which of the following is (are) true of an existing Medical Savings Account (MSA)? I. An MSA may receive contributions from any person, including an employer or family member, on behalf of an eligible person. II. Contributions by the eligible individual are deductible even if that person does not itemize. III. Employer contributions are not included in the eligible person's income. IV. Both the employer and eligible person may contribute to the MSA in the same tax year. A) I B) II, III C) I, II, IV D) I, III, IV E) All of the above

B) II, III Statements I and IV are wrong because the only parties that can contribute are the eligible person or the employer - but not both in the same year. Contributions by the eligible individual go on the front of the 1040 (adjustments to income).

Which of the following describes a Health Flexible Spending Account? A) It is an employer-funded plan that makes available to employees a fixed pool of money to reimburse for qualified medical expenses (not taxable to employees). B) It is an option under a cafeteria plan through which employees may defer salary on a pretax basis into an account created exclusively for reimbursing or paying the qualified medical expenses of the employee. C) It is a trust created exclusively for the purpose of paying for the qualified medical expenses of the account holder. The account holder may be any individual covered by a qualifying high deductible.

B) It is an option under a cafeteria plan through which employees may defer salary on a pretax basis into an account created exclusively for reimbursing or paying the qualified medical expenses of the employee. Answer A is a Health Reimbursement Arrangement (HRA). Answer C is a Health Savings Account (HSA). Answer B is an FSA.

Neil Aldrin, a retired 70 year old single male, comes to see you, a CFP® practitioner, about planning for his granddaughter. He has total assets of $5,500,000. $3,500,000 is invested of which $2,500,000 is in his Traditional IRA. He is upset about having to start RMD's in a few years and thinks the IRS are crooks. Neil has no debt but he is always interested in a good time and fast cars. He is as healthy as a 35 year old and expects to live past 100. He likes to be able to sell stocks and pay cash for whatever he comes across at the drop of a hat. His auto collection, while not up to Jay Leno's, is fairly extensive, with hotrods, sports cars, uni body pickups, and choppers. He wants his collection to go to a museum when he is gone. In his 20's, Neil was a test pilot/astronaut for NASA's Apollo missions in the early 70's. He flew the prototype lunar landing craft to determine whether or not man could sustain controlled flight in space. At one point, he and the craft were 49 miles away from the command module for a period of 5 hours and 200 orbits around the earth. Had he not been able to reconnect with command module for reentry, he would have literally been "toast". This type of daredevil attitude has carried over into his investment philosophy. He has never had an advisor and has always picked his own stocks. He is and has always been allocated 100% equities. His son, Buzz, is also a risk taker, but in a different way. He has never had a job other than being a backup cowbell player in various regional cover bands. Buzz plays a mean cowbell. However, his only real accomplishment is having a daughter, Eva, age 8. It is Eva that Neil wants to take care of. Neil is not convinced that Buzz's career path will lead to stardom. He also doesn't trust Buzz with money and did setup a revocable trust years ago. The trust has $1,000,000 in stocks. What do you recommend that Neil do? A) Recommend he convert his Traditional IRA into a Roth IRA to save income taxes on future withdrawals and avoid having to take RMD's. B) Recommend a complete cash flow analysis and considering various options concerning income, gift and estate taxes C) Recommend that he reduce his exposure to the stock market risk due to his age and time horizon. He cannot afford to take a substantial loss. D) Recommend that he start taking distributions from his IRA now and fund a life insurance policy inside of an ILIT for Eva's ultimate benefit.

B) Recommend a complete cash flow analysis and considering various options concerning income, gift and estate taxes B is right because his ability to live the lifestyle he desires should take precedence. Nothing is mentioned in the question about what his needs are. Option A would produce significant taxes. Option C is probably good suggestions too but if we don't know his retirement needs, we don't know his required rate of return. Option D is also not a bad suggestion considering the size of his estate and his health. Yes, some what suggestive but remember this is what the exam questions will be like. On the exam, you must the best decision and move on.

Aunt Patricia, wanted to leave her million dollar estate to her two favorite nieces, Trisha and Patsy, who are both now in their mid-thirties. Trisha has always demonstrated that she handles money well. However, Patsy has a history of credit card along with other debt and declared bankruptcy three years ago. Considering the situation, Aunt Patricia left half of her estate to Trisha by will and instructed that Patsy's half be placed in trust with Trisha as trustee. Under the trust instrument, Patsy's children are remainder beneficiaries. Trisha is given sprinkling power in the trust and may distribute income and principal as she sees fit. Patsy contacts Tricia asking for a withdrawal of $50,000 to be used to install a lighted stable and corral in the substantial grounds surrounding her Kentucky vacation home. Which of the following represents the most appropriate response from Trisha? A) Make the distribution as Patsy requested B) Refuse to make the distribution. C) Arrange for Patsy to borrow the $50,000 from the trust under reasonable terms. D) Speak with the family's clergy to determine the morally correct decision.

B) Refuse to make the distribution. In light of fiduciary responsibility to the remaindermen, discretionary power, and an understanding of the intent of the trustor, Tricia should refuse the request. Having Patsy borrow from the trust would not be appropriate, and given Patsy's history, could be risky for the trust. This is a matter of law rather than a matter of faith.

The term "self insurance" applies to which one of the following methods to avoid financial risk? A) Assumption of risk B) Retention C) Transference D) Avoidance

B) Retention Assumption of risk is a good answer; however, the proper use of self-insurance applies to risk retention. Self insurance is not transference normally, no insurance is purchased. The company assumes the risk.

Patricia Cox owns a condo in Florida. Due to a hurricane, the condo roof was damaged. She has an HO-6 unit owners policy. Does she have coverage if the condo owners association assesses her for the deductible under the association policy? A) She has no benefits under Coverage A. B) She has named-perils benefits under Coverage A (limited). C) She has 10% of Coverage C benefits for Coverage A. D) She has benefits under Coverage C.

B) She has named-perils benefits under Coverage A (limited). According to one textbook, the HO-6 policy does provide $5,000 for Coverage A on a named-perils basis. Most HO-6 policies have a $1,000 payment for loss assessment (also under coverage A). This is the best choice. The other answers are wrong.

John, age 69, and Mary, age 66 (married) have been collecting Social Security benefits of $1,500 and $750 per month. If he dies, what will happen to her benefits? A) She will get nothing. She gets 1/2 of his benefits. B) She will get his current benefit. C) She will get $1,250 per month. D) She will get $2,250 per month.

B) She will get his current benefit. She will get the greater of her benefits or 100% of his benefits.

When dealing with a client's estimated unequal cash flows from a potential investment, what is the major difficulty that you, as a financial planner, may encounter? A) Whether to use time value of money B) What discount rate to use C) Whether the investment under consideration should be rejected if the NPV is a negative number D) How the IRR will be reinvested

B) What discount rate to use Answer C is a good answer. The difficulty is in determining what discount rate (client's required rate of return) to use for the investment.

Scott Harding died recently (at age 74), with a taxable estate of roughly $8 million. Some years ago he had seen an estate tax attorney and completed an extensive estate plan with a variety of trusts. Among his assets is a stone-constructed New England farm house having a current fair market value of $2 million. Scott's will bequeathed the home to Clarisse who, due to authoring a best-selling mystery novel, is worth approximately $20 million in her own right. Nevertheless, when Scott died, Clarisse, (under the guidance of her tax advisor), did not want to own the home outright, fearing it would become part of her already substantial gross estate. However, she wishes to live in the home where she can continue to write and enjoy her grandchildren for the rest of her life . The home had been in the Harding family since its arrival in America in 1789. At the time of his death it was owned fee simple by Scott. There is great sentiment associated with the home: Scott and his wife, Clarisse (age 66) were married in the home and raised their two children Rebecca and Jonah (now adults) there. What technique, if any, will accomplish Clarisse's goals? A) Clarisse cannot have her cake and eat it too: If she disclaims ownership of the home, she may not live there without paying fair market rent. B) With a proper provision in Scott's will, if Clarisse disclaims ownership of the home, it can be transferred to a disclaimer trust, the terms of which permit Clarisse to live in the home for her lifetime. This would be a family trust. C) With a proper provision in Scott's will, if Clarisse disclaims ownership of the home, it can be transferred to a disclaimer trust, the terms of which permit Clarisse to live in the home for her lifetime. This would be a marital trust because Clarisse has a life interest in the home. D) A qualified personal residence trust (QPRT) should accomplish Clarisse's dual goals of not including Scott's home in her gross estate and the right to live in the home for her entire lifetime.

B) With a proper provision in Scott's will, if Clarisse disclaims ownership of the home, it can be transferred to a disclaimer trust, the terms of which permit Clarisse to live in the home for her lifetime. This would be a family trust. Many well written wills include disclaimer trust provisions which give the surviving spouse the ability to put specific disclaimed assets into the trust by disclaiming ownership of a portion of the estate. Disclaimed property interests are transferred to the trust, without being taxed. Provisions can be written into the trust that provide for regular payouts from the trust to support survivors, or in the case of the Hardings, the right to occupy (but not own) property. The trust can also be written so that surviving minor children can also be provided for, as long as the surviving spouse elects to disclaim inherited assets, passing them on to the trust. To keep the assets from being included in Clarisse's estate, the trust would have to be a family trust rather than a marital trust. If a disclaimer trust is used, the full extent of the tax planning occurs upon the death of the first spouse. At that point, the surviving spouse can either accept the trust assets or disclaim them. If he or she disclaims them into the disclaimer trust, the trust will function like a credit shelter trust that will "shelter" the assets from inclusion in the surviving spouse's estate. But if there is no tax reason to use credit shelter planning, the spouse can simply receive the assets outright. This allows tax-planning flexibility without creating unnecessary complication. Using answer D will mean for the life of the QPRT it could be brought back into her estate. It is not a bad answer. With Answer B, the house uses the exemption. Suggestion: The questions are not too long to read. I would recommend reading it first.

Ken, age 65, is eligible for Medicare. He would like to delay enrolling in Medicare until he retires. He asked you if he can continue to be a participant in the company's HSA. What was your response? A) Because he is eligible for Medicare, he can no longer be eligible for the company's HSA. B) Yes, he can continue to participate as long as he does not enroll in Medicare A and B. C) Yes, but only if he doesn't enroll in Medicare Part B

B) Yes, he can continue to participate as long as he does not enroll in Medicare A and B. An individual is "entitled to benefits under Medicare" only when the individual is both eligible for and enrolled in Medicare. Enrollment in either Medicare Part A or Part B will disqualify an individual from contributing to an HSA. But mere eligibility does not preclude HSA contributions. NOTE: You are not automatically covered for Medicare at age 65. You must enroll by contacting Medicare.

Mr. Able created a $1,000,000 GRAT. He reserved a 5% annuity for a period of 20 years. If the grantor retained interest is $563,165. What is the amount of the taxable gift? HINT: Think simple A) $420,835 B) $425,835 C) $436,835 D) $549,165 E) $563,165

C) $436,835 The amount of the gift ($1,000,000 gift - 563,165 retained interest) is $436,835. This is a gift of a future interest, not a gift of a present interest. This is why Answer A is wrong ($16,000). The taxable gift is $436,835

If the beta of a stock is 1.2, the risk free rate is 5%, and the return on the market is 8% then, according to the CAPM, what is the required return for the stock? CAPM Formula: r = rf + (rm - rf)β It will be given on the exam - see formula sheet. A) 3.6% B) 6% C) 8.6% D) 9.6% E) 13%

C) 8.6% The CAPM uses the SML (required rate of return) formula: r = rf + (rm - rf)β = 5% + (8% - 5%)1.2 = 5% + 3.6% = 8.6%

Which situation shown below best describes financial infidelity? A) When La Verne has weekly appointments with Fabio, the pool boy, at the Nap Rate Motel, she always pays for the room. B) Jay named his daughter as beneficiary of his SEP IRA. C) Carmen's husband doesn't know that she recently purchased fine jewelry worth $20,000, charging it on their joint Visa card.

C) Carmen's husband doesn't know that she recently purchased fine jewelry worth $20,000, charging it on their joint Visa card. Carmen is hiding substantial expense from her husband. This clearly indicates financial infidelity. Nothing in Answer B indicates that Jay's wife is unaware that he named his daughter as beneficiary of his SEP account. Mary and Paul got carried away with spending for their daughter's wedding but there was no concealment between them. Answer A is a different type of infidelity.

Harry Potter bought a $25,000 single premium deferred annuity 30 years ago at age 30. Now approaching retirement he is trying to decide if he should take the cash value ($110,000) as a lump sum or annuitize the policy over a 20 year single life expectancy of $725 per month. When he enters his retirement years his tax bracket will drop to 12% and he is concerned about income. What would you recommend if he feels he can invest the lump sum and achieve a 6% after-tax return? A) He should take a lump sum and pay the tax B) If he takes a lump sum he will have to pay the tax plus a 10% penalty (annuity rules), he should annuitize. C) He should annuitize the contract. D) He should take a lump sum because all he will get is 20 years of payments.

C) He should annuitize the contract. Well, if he takes a lump sum Answer A he will pay 12% on $85,000 ($110,000 - 25,000 basis) or $10,200. ($110,000 -10,200) x 6% = $5,988/year $725 x (1 - 12.00%) = $638.00 $638.00 x 12 = $7,656/year Answer C has a higher net of tax payout. Answer B is wrong. He is over 59½ so there is no 10% penalty. Answer D is wrong as the payments continue after 20 years, but are 100% taxable.

Which statements regarding a health FSA are correct? I. It may receive contributions from an eligible person. II. Allowable contributions are not subject to FICA. III. Allowed contributions are not included in income. IV. Reimbursements from the health FSA used for expenses are not taxed. A) I, III B) I, II, IV C) I, II, III D) II, IV E) III, IV

C) I, II, III Statement IV is wrong because it does not specify "qualified" medical expenses.

Which of the following organizations would be considered public charities (50% organizations)? I. Education organizations with regular faculty and curriculum and regularly enrolled students II. Hospitals III. The Rotary Club (A not-for-profit), which raises money for public causes IV. The United Way V. Public libraries A) All of the above B) I, II, III, IV C) I, II, IV, V D) II, IV E) III, V

C) I, II, IV, V Rotary is a not-for-profit. It isn't a public or private charity. The Rotary Club would have to obtain IRS approval to receive tax deductible contributions.

Carrie Adams, a 45 year old single mother of three boys, ages 10, 12 and 14, has come to you seeking financial planning advice. Carrie's husband, Adam, a self-employed general contractor, passed away eleven-months ago in a tragic bulldozer accident. Adam's business had been fading a bit due to spending more time with his children. His business was a sole proprietorship, sub contracting out labor on various projects. Adam spent the majority of his time and financial resources on growing his business. He was not focused on saving for the future, rather living for the moment. He believed he would be positioned for a huge growth in business, leading him to an early retirement. Unfortunately that did not happen and Carrie is stuck to figure things out. On Carrie's belief that Adam's death could have been avoided, she hired an attorney to sue the real estate developer for wrongful death. A disgruntled worker for the real estate developer testified as to the company's lack of safety procedures. This lead to a quick decision and Carrie was awarded compensatory damages in the form of a twenty-year period certain annuity paying her $2,500 per month (this was a dollar figure determined by a court hired economist based on a variety of factors). Adam did not have any life insurance, and the little bit of savings they did have is dwindling by the moment. Carrie has been out of the work force for the last fourteen years, concentrating her time on taking care of the home and homeschooling her children. Her three boys, Andrew, Anthony and Alex, were enrolled in public school shortly after their father passed. Prior to the birth of Andrew, the oldest, Carrie worked as a paralegal. Since Adam's income was not always stable, Carrie, conservative by nature, always kept her paralegal licenses up to date. As a result, she was able to find full time work quickly and next week Carrie begins work at a mid-size law firm - the same firm that represented Carrie against the real estate developer. Her salary is $45,000 per year. Her employer does provide some benefits to employees. The law firm provides a high deductible HSA group health insurance, although premiums are paid 100% by the employee for themselves and family. Retirement benefits are also provided through a 401(k) plan and a free annual financial plan conducted by Victor Vector, a CFP® professional. With the lawsuit and her job, she is not too worried about paying the bills. Her concerns are (in no particular order), saving for retirement, providing a college education for her children, enrolling the boys into a private school as soon as possible (she doesn't like public school) and paying off her mortgage. During your first meeting you discover that Carrie does not have any life insurance, disability insurance or estate planning documents. Left over from Adam's general business account is $30,000, which she transferred to her personal checking account. She has spent some of it over the last eleven-months for legal fees and to pay monthly living expenses. Carrie was the beneficiary on Adam's SEP-IRA. The current value is $150,000. She estimates her monthly essential expenses to be $4,500 per month. Given Carrie's situation, which of the following recommendations would you make as a CERTIFIED FINANCIAL PLANNERTM professional? I. Purchase a maximum amount of 20 year term life insurance that she could be underwritten for by a top rated life insurance carrier. II. Contribute to a Coverdell ESA and their state's pre-paid 529 plan to allow her to fund current private school tuition and future college cost of tuition. III. Contribute to her employer's 401(k) plan with a 50% match up to 12% of her salary (5-year vesting schedule). IV. Purchase a $400,000 Variable Universal Life Insurance policy, contributing extra premium (non-MEC) to the policy to help with retirement savings. V. Establish a monthly savings plan to build up her emergency fund to an adequate level. A) II, IV, V B) I, V C) I, III D) III, IV, V E) I, II, V

C) I, III It is important that Carrie takes care of her three children. In the event of her untimely death, they could be left destitute. This question does not mention anything about a wealthy family or others that could take care of her children in the event she was to pass away. It is vital that Carrie obtain life insurance. Of the choices including life insurance "Roman Numeral I", 20 year term life insurance, is a better choice than "Roman Numeral IV", Variable Universal Life. Term life insurance will cost her less in monthly premium and she should be concerned more about her children's well being than tax deferral and supplemental savings. Providing maximum death benefit at a lower overall premium is a better option than cash value life insurance, at this point in her life. Carrie's gross income is $75,000 per year. Considering her situation - single with three children - her income tax liability is minimal. Her monthly net income will be roughly $5,500 per month. Considering her essential expenses are $4,500 per month, she does have a little bit of room to save. It is more important for her to save for her retirement than for her children's education. Both saving for retirement and education are goals of hers, however, saving for retirement is more important at this stage in her life, especially considering the generous 401(k) match. Thus, "Roman Numeral III" is a better choice than "Roman Numeral II". Plus, saving 12% of her salary in a 401(k) is $5,400, or a ~$400 net monthly "loss" of income. Since her essential expenses are $4,500, this still gives her an extra ~$600 per month on discretionary expenses. Finally, her children can always get loans for college. There has yet to be a retirement loan created...yet. Lastly, Carrie had $30,000 in her personal checking account. Even though she is a single income earner, she does have a second source of income, the 20 year period certain annuity. An adequate emergency fund would be at least three months of her essential expenses (3 months x $4,500 = $13,500). The best answer is C. Suggestion: I would have read the answers first. This is a very long question.

Sally Single works as an employee for a large company. The company has recently reduced health benefits and increased deductibles in the company health insurance plan. Sally wants to know what kinds of medical care costs not covered by her insurance are deductible subject to 7.5% of AGI. I. Dental x-rays II. Athletic club expenses III. Bottled water bought to avoid drinking fluoridated city water IV. Divorced spouse's medical bills V. Eyeglasses A) All of the above B) I, III, IV, V C) I, V D) III, IV, V E) IV

C) I, V Medical expenses include the cost of diagnosis, cure, mitigation, treatment, or prevention of disease or any treatment that affects a part or function of your body.

During the year, Fred Smith had the following expenditure for his rental house. The expenditures are listed below. Fred has a full-time job and files a Schedule E (active participation) for the rental house. Which of the expenditures must be depreciated rather than deducted as an expense on his Schedule E? I. Replaced a screen in a window II. Replaced the air conditioning system III. Built a swimming pool IV. Installed a new water heater V. Hired a lawn service to cut the grass A) I, II, V B) I, IV, V C) II, III, IV D) III, IV E) III

C) II, III, IV Items I and V are expenses. The others are improvements even though II says replace and IV says install. There is no list of what are expenses and what are improvements.

A client approaches you on an investment. She is going to invest $100,000. She tells you the investment will produce a series of unequal cash flows and has an estimated future value. What method would you use to calculate her return? A) Time weighted return B) Arithmetic mean return C) IRR D) NPV

C) IRR The arithmetic mean is a simple average. The time weighted (geometric) return method can only be used for returns, not cash flow/future value. NPV cannot be used because no required rate of return was given. You must input the required rate of return to calculate NPV. IRR is used to calculate her return.

John is a CFP® practioner who is establishing a relationship with forty year old Annie Wong in California. They have met once, and in that meeting John was able to establish and define their relationship and gather all of Annie's financial data. While in the process of preparing for their second meeting and analyzing the client data he received an email from Annie stating that she had been reassigned to a 6 month project in Japan and will not be able to make their next appointment in which John had planned to present his recommendations to her. Annie would still maintain her California address and residency and US citizenship and intends to return to live in California when the assignment is completed. Since she felt it would be difficult to meet in person or over the phone she has asked John to email her the list of recommended investments for her portfolio. John obliges her request, finishes analyzing her situation and sends her an email containing a list of the investments he recommended for her and the percentages she should place in each investment. A few days later John received another email from Annie in Japan stating that she liked the recommended growth portfolio and had gone online and executed the trades on her own in her account. She thanked John for working with her unique situation and planned to meet with him again some time after she returned from the overseas assignment. Has John violated any of the CFP® board's practice standards in this scenario, or is this a suitable approach to her situation? A) John may have breached the implementation step of the planning process because Annie executing the trades online is not a suitable form of implementation. B) John may have breached the monitoring step of the planning process because there was no discussion with Annie about the monitoring of her investments or next steps. C) John may have breached the presentation step of the planning process because emailing a list of funds does not meet the necessary presentation guidelines. D) Assuming John followed all FINRA and SEC guidelines regarding working with international investors, this is a suitable method of implementing Annie's plan.

C) John may have breached the presentation step of the planning process because emailing a list of funds does not meet the necessary presentation guidelines. Answers A and B, although possibly true, are not the root cause of the issue here. The main problem John has created here is that in doing this "favor" for Annie, he has not met the guidelines of presenting a plan set forth by the CFP Board which requires that the CFP® practitioner shall communicate the factors critical to the client's understanding of the recommendations. These factors may include but are not limited to material: personal and economic assumptions; interdependence of recommendations; advantages and disadvantages; risks; and/or time sensitivity. Sending a list only of the recommendations clearly does not explain these factors to Annie.

Robin holds both a FINRA Series 7 General Securities Representative's license, and she is a CFP® professional. She has been a financial planning practitioner for several decades and enjoys a cordial relationship with Max Fox, who is one of her favorite clients. Max's trading account has been under a maintenance margin call for the past two days. Max is fishing in Costa Rica and not easy to reach. From reading her employing broker-dealer's policy manual, Robin knows that in one more business day the firm will liquidate Max's position. She also knows that this will make Max upset and angry. From the list of choices, what is the best course of action for Robin? A) Do nothing; the firm's rules are there to protect against debits in customer accounts B) Write a check from her personal funds to satisfy the maintenance call. Max is obviously trustworthy and would appreciate the gesture C) Make every effort to communicate with Max in a timely manner D) Call Max's (adult) son to explain the immediacy of the situation and seek guidance from him on how to get hold of Max.

C) Make every effort to communicate with Max in a timely manner While the representative/planner is to serve the interest of her client(s), she must operate within rules and ethical principles when doing so. Writing a check to cover her client's maintenance call would be considered lending a best and commingling at worst and violates the CFP® Code of Ethics. Discussing the margin call with Max's son violates Confidentiality. Trying to reach Max shows a higher fiduciary (good faith) standard than not even trying to communicate and doing nothing. It doesn't say he isn't reachable. She has one more business day to reach him.

Alex and Susan Halton have been married for 5 years and each has children from a prior marriage. Alex is self-employed as a homebuilding consultant with earnings after all expenses of $150,000/yr. and plans to work 5 more years, while Susan is retiring from Verizon this year with pension income of $60,000/yr., where her salary was 89,000/yr. Susan exercised 50 incentive stock options this year with an option price of $20, when the stock was trading for $40 per share. Also, Allan purchased 3 business use vehicles for his consulting practice in January for $45,000 each and plans to forfeit any "special depreciation allowance" or section 179 expense, and instead depreciate them using 5yr MACRS. Alex and Susan want to invest some of their liquid assets totaling $50,000 into investments for income, and have asked for your help. They already have saved significant qualified retirement plan assets in excess of one million dollars in well diversified mutual fund portfolios. What should you recommend? Hint: The CFP Board gives you income tax charts. Are they in the 10-12% bracket? Are they in a 37% bracket? You have to make decisions on this exam. A) Recommend investment grade corporate bonds currently yielding 6%. B) Recommend investment grade AMT private activity municipal bonds for several of the states power companies currently yielding 4.5%. C) Recommend state general obligation public purpose municipal bonds yielding 4.6% . D) Recommend Verizon common stock, currently trading at $44 per share, since you know Susan will love that idea based on her tenure with her lifelong employer, and because she mentioned that Verizon pays a good dividend of $2.06 per share annually.

C) Recommend state general obligation public purpose municipal bonds yielding 4.6%. The correct answer is "C" because it has the highest yield for their tax situation, and is an income investment. There is no mention of maturity dates, so that should not be a factor to consider. There is no mention of risk tolerance, so no judgment can be made on that. The questions states "investments for income" not "growth and income", so stocks would not be suitable based on the client request. A. Corporate bonds at 6% would yield an approximate after tax income of 4.56% (24% tax), which is lower than the tax free yield of 4.6% B. AMT could be triggered based on the ISOs exercised by Susan and the MACRS depreciation for Alex, therefore, a private activity AMT bonds would not be suitable for them, as they would end up paying AMT tax of around 26% on it bringing the yield to about 3.33%. C. The taxable equivalent yield of the 4.6% municipal bonds should be approximately 6.05% (24% tax) by dividing by the factor of .76 . It is the best income for them. D. Just because you know a client will like an idea, does not make it ethical. A common stock should not be thought of as an "income" investment because of the growth component, and the added volatility from equity investments. Even though, the yield ($2.06/$44) or 4.68% for a qualified dividend would still be lower than the G.O. muni bond. (At a 15% cap gain rate the after tax income is 3.98%)

An existing client calls you, a CFP® professional, with a question about a topic you aren't very familiar with. She tells you her husband served in the military decades ago and is now receiving care in a nursing home. She read a brochure at this new facility about a VA benefit called Aid and Attendance that could pay her around $1,500 per month as the spouse of a serviceman who is receiving skilled nursing care. She would like you to help her qualify for the benefit. You tell her that you will do some homework and arrange a meeting with her the following week to discuss their situation further. After doing some research, you discover that there are very stringent income and asset restrictions to qualify for benefits. Since they have been clients for years, you know they have a net worth of around four million dollars. In your meeting you should: A) Explain the income and asset thresholds that are well lower than their current situation. Help her understand that this benefit is not intended for people in their financial position and recommend they not pursue the benefit further. B) Tell your client you aren't an expert in this matter and put her in touch with an attorney with whom you have worked the past whom you know to be a military veteran. C) Review options for how the client may effectively reduce income and gift assets to qualify for the benefit. D) Recommend she purchase long term care insurance and review their current retirement and investment objectives for other options to meet their needs.

C) Review options for how the client may effectively reduce income and gift assets to qualify for the benefit. In D, you never address her objective and discuss options on how to qualify; you only give recommendations apart from it. A is a good answer however you should let her determine if she is willing to go to lengths to qualify, you shouldn't be making the decision for her by dismissing the option. B would be a great answer but it is unclear if the attorney is competent in the subject. In this meeting, you should review the client's options as she requested, answer C. Once that is done you may then decide to advise a more suitable solution (Answer D). If you answered B I am not saying you are wrong. This is a practice question. (Subjective)

What is the advantage between taking excessive income as a stockholder from a corporation or an S corporation if the corporation is very profitable? A) Taking earned income as compensation from a corporation will qualify the stockholder to ultimately get more Social Security benefits. B) Taking earned income as compensation from a corporation will qualify the stockholder for a large retirement contribution. C) Taking limited income from a S corporation will reduce FICA and FUTA taxes because the remainder of income will be unearned income. D) Taking limited earned income from an S corporation and the remainder as unearned income will reduce corporate taxes.

C) Taking limited income from a S corporation will reduce FICA and FUTA taxes because the remainder of income will be unearned income. In answer A and B there are salary caps that limit what qualifies for Social Security benefits and retirement contributions. But compensation is subject to unlimited Medicare taxes (disadvantages). That is why Answer C is a better answer. Unearned is not subject to Medicare. Answer D is incorrect, an S corporation does not pay corporate taxes (conduit entity). You may have chosen Answer D because it is an advantage for an S corporation owner. Yes, S owners reduce their wages to pay less FICA taxes by taking the maximum unearned income. This is why many people do an S corporation - avoid FICA taxes. But that is not what Answer D says. You have to answer the question as written.

Toby, age 72, has been taking distributions from his qualified plan. Now, to his dismay, he has been sued and lost in court. His only real asset is the qualified plan distributions. Can the plaintiff collect against his qualified plan distributions? A) No, ERISA forbids "alienation of plan benefits." B) No, Toby can file for Chapter 7 bankruptcy and keep his plan benefits. C) Yes, but only for a portion of the distributions D) Yes, because that is his only asset

C) Yes, but only for a portion of the distributions The "anti-alienation" provision only applies during the accumulation period, not after the account goes into pay status. Once the distributions are paid out, they become an asset of the participant, subject to any creditor's claims that may be pending when the benefits are received.

Tom is a participant in an HSA. He wants to pay for LTC premiums with distributions from an HSA. Is he allowed to do that? A) No, the HSA can only pay for LTC expenses, not premiums. B) No, the HSA can only pay for qualified medical expenses. C) Yes, the HSA can pay for qualified LTC premiums (age-based). D) Yes, the HSA can pay for all LTC premiums and expenses.

C) Yes, the HSA can pay for qualified LTC premiums (age-based). This is the best answer. The amount of qualified long-term care insurance premiums that constitute qualified expenses is allowed. However, the LTC premiums are still limited to age-based limitations (as adjusted annually). Answer B does not specifically answer the question. This is an HSA not an FSA. Premium payments are not allowed in FSAs but are allowed in HSAs.

A client has investment income of $500,000. The client takes a loan of $50,000 from a bank to buy 20 acres of land for investment purposes. The real estate will be the collateral for the loan. The loan cost for the year is 10% or $50,000. Is the interest tax deductible? A) No, it is a passive investment (passive loss). B) No, the client is not materially participating in real estate. C) Yes, up to investment income. D) Yes, but the client has to have an AGI of at least $50,000.

C) Yes, up to investment income. Investment interest ($50,000) is tax deductible up to investment income ($50,000). Investment interest is any interest incurred to purchase property that is held for investment, such as raw land.

A corporation purchased a whole life insurance policy for the life of a key employee. The corporation was both the owner and beneficiary under the contract. The corporation did not deduct the premium but instead listed the premiums paid as an asset on its financial statement. The corporation borrowed from the insurance company and various banks using the policy as collateral for the loans. Today, the corporation surrendered the policy for a gain. How will the gain be treated for income tax purposes? A) As a long-term gain because a termination of a right with respect to property is a capital asset of the taxpayer B) As a long-term gain because life insurance contracts are considered capital assets C) As a long-term gain because the cash value growth was not treated as ordinary income but it is treated as a capital asset D) As ordinary income E) As a return of basis, a nontaxable event

D) As ordinary income Life insurance surrender gain above basis is always taxed as ordinary income.

Judith Everet, age 55, has been your client for years. Divorced 10 years ago with generous marital settlement, very conservative with a low risk tolerance. Asset portfolio for past years since 2007 market downturn has been 70% in bonds, 20% in money market, and 10% in growth and income funds. As a result of inflation and life style changes, travel to Europe and Far East, her portfolio has gone down. She has taken all the income. She comes to you today to take 40% and invest it in emerging growth. She is taking 20% from bond and all of the money market as her source of investment. What should you do? A) Make the investment B) Make the investment document her decision C) Do not make the investment, explain to her that the risk is beyond her profile. D) Do not make the investment, educate her on the risk associated with emerging growth.

D) Do not make the investment, educate her on the risk associated with emerging growth Very tough question/answer. All the answers are good, but educating her is best.

Mr. Rich is gifting $100,000 cash to a political organization (an American political party). How will the gift be handled? A) If he has an AGI of at least $200,000, he can deduct the $100,000. B) He will have to declare a taxable gift of $86,000. C) If he has an AGI of at least $333,333, he can deduct the $100,000. D) Gifts to political organizations are exempt from gift tax.

D) Gifts to political organizations are exempt from gift tax. Answer D is true because Mr. Rich is gifting to an organization not a charity. There is no tax deduction with a gift to a political party.

Which statement(s) is(are) true about an existing Health Savings Account (HSA)? I. An HSA may receive contributions from any person, including an employer or family member, on behalf of an eligible person. II. Contributions, other than employer contributions, are deductible on the eligible person's individual tax return only if that person itemizes. III. Employer contributions are not included in the eligible person's income. IV. Both the employer and eligible person may contribute to the HSA in the same tax year. A) I B) I, II C) II, IV D) I, III, IV E) All of the above

D) I, III, IV Statement II is wrong because the contributions are deductible on the eligible person's tax return even if that person does not itemize. They go on the front of the 1040 (adjustments to income).

Larry is disabled due to a job-related injury. He is being paid benefits under workers compensation, Social Security, and his private disability plan (employer paid). Which of the disability benefits could be taxable? I. Workers compensation disability benefits II. Social Security disability benefits III. Private disability plan benefits A) All of the above B) I, II C) I, III D) II, III E) II

D) II, III If his MAGI is above certain levels, the Social Security benefits could be taxable. The private disability plan benefits are taxable because the employer paid the premiums. It does not say it was a taxable bonus. If the question said Larry paid the premiums on the private disability plan, then what would be the answer? Answer E

Mark Spout created an irrevocable trust for the benefit of his dependent children. Mark named the local bank as trustee of the trust and authorized it to invest in stocks, bonds, and negotiable certificates of deposit. Included in the investment authority is the right to use trust income to purchase insurance on Mark's life. All funds are currently invested in high-yielding bonds paying 7% semiannual interest on a par value of $100,000. Twenty-five percent of the bond investment income is being used to pay the premium on a policy on Mark's life. Which taxpayer must pay tax on the income of the trust and why? A) The bank because of its broad authority as trustee B) The children because the income is paid by them C) The trust because it is irrevocable with no benefits to grantor D) Mark because of the grantor rules

D) Mark because of the grantor rules If any portion of the trust income is, or may be, used to purchase insurance on the life of the grantor or grantor's spouse, then the trust is a grantor trust. The trust is/was also used to benefit his dependent children (support). 25% is to purchase insurance and the remainder is used for support.

Gail Goodrich, single, owned a small corporation, GG, Inc. Due to business reversals, she had to close the business, and the stock became worthless. Which of the following is true if the loss is $120,000? A) If she meets the requirements of Section 1244, she can take an ordinary loss of $100,000 and a capital loss of $3,000. The remaining $17,000 is a carry-forward loss. B) She can take a $103,000 short-term loss. C) She can take a $53,000 long-term loss. D) None of the above

D) None of the above Answer A is wrong because she is single, not married. Section 1244 allows for a $100,000 ordinary loss for married filing jointly and a $50,000 ordinary loss for single persons. Answers B and C are wrong because we do not know how long she was in business. The correct answer for single is $50,000 ordinary loss plus $3,000 capital loss and a $67,000 carry forward.

Larry Towne and Robert Dunlap formed a service related business about 10 years ago. At first they run it like a partnership until they got sued and the liability issues of a partnership nearly caused them to file for personal bankruptcy. After the issue was settled they converted to an S corporation. When it became reasonably successful, they changed it to a regular C corporation. Now Larry is 49 and Robert 47 are considering adding some type of retirement plan. Profits have stabilized along with their salaries (37% personal bracket). They both feel they want to retire in about 20 years. They have mixed emotions on how to fund the contributions. They think the employees should defer something but the company should provide contributions to the maximum allowable. They would like a plan which will help them retain employees. Most of the employees are in their 30s and making between $60,000 to $90,000. They have 6 employees who would be eligible and they feel the company could contribute $150,000. Which plan would you suggest? A) 401(k) SIMPLE B) SIMPLE C) Stock bonus 401(k) D) Profit sharing 401(k) E) Defined benefit integrated with Social Security

D) Profit sharing 401(k) With answer A, they would be limited to a deferral of $14,000 and 3% of the salary cap (305,000). With Answer B, it is slightly better with a deferral of $14,000 and 3% of $466,666. There is no indication that they want to share the company ownership with the employees (stock bonus). The profit sharing with a deferral will be the most flexible and effective. Defined benefit does not fit.

A client, T.T. Banks, wants to meet with you. His son, Tubby, finally, graduated from college. T.T. wants to establish a brokerage account for his son to encourage him to start earning money and invest it. He will open his son's account with a $6,000 gift. How should you proceed? A) Open the account and place the money in a Roth IRA. Since it is a contribution, Tubby can remove the money tax-free. B) Request a joint meeting with T.T. and Tubby to start the planning process. C) Open the account and place the money in a money market account until Tubby gets a job. D) Require Tubby to come in to complete a cash flow analysis and risk questionnaire before you can open the account do any investments.

D) Require Tubby to come in to complete a cash flow analysis and risk questionnaire before you can open the account do any investments. You cannot do a Roth. Tubby has no earned income. Answer C is not bad. Answer D is the first step in the financial planning process. Should T.T. be involved in the process since he is making the gift. Tubby will be the client.

Tammy is getting a divorce. As part of a property settlement, she is getting 50% ($250,000) of her husband's IRA ($500,000). Out of the settlement, she needs to buy a car ($20,000) with cash. What should she do if her AGI is less than $100,000 this year (single)? A) Take a distribution for $20,000 and roll over $230,000 into her own IRA. The $20,000 will be subject to ordinary income tax but no 10% penalty (QDRO exception). B) Roll the $250,000 into her own IRA. If she pledges $20,000 of the IRA to make the car loan, only the loan interest would not be deductible (consumer interest). C) Roll the $250,000 into a Roth IRA and then withdraw $20,000 to buy the car. Only the $20,000 would be subject to ordinary income tax. D) Take a 60-day distribution for the whole $250,000 and buy the car. Then, before the 60 days are up, she could roll all available funds into her IRA and pay ordinary income tax and a 10% penalty on the non-rollover money.

D) Take a 60-day distribution for the whole $250,000 and buy the car. Then, before the 60 days are up, she could roll all available funds into her IRA and pay ordinary income tax and a 10% penalty on the non-rollover money. Answer A is wrong because this isn't qualified plan money. QDROs only apply to qualified plan distributions. Answer B is wrong because pledging the IRA to make a loan makes the IRA subject to income tax and a 10% penalty. Doing Answer C would make the whole $250,000 subject to income tax and the $20,000 subject to a 10% penalty. I think Answer D is the best answer. During the 60 days, she might find a cheaper car or find more liquid assets to reduce the taxable event. This is the only possible answer.

Mary Tuttle, a single mother, has a dependent child, John. John tries to help out his mother. This year he netted $4,200 doing odd jobs receiving 1099s and also received $350 in interest on his savings account. No income tax was filed. Did John have to file an income tax return? A) No. His standard deduction, $12,950, covers his earned and unearned income. B) Yes, his exemption is less than the total of his earned and unearned income. C) No D) Yes

D) Yes Answer A is actually earned $4,200 plus $400. However, it does not answer the question. If you do freelance work as an independent contractor, you are self-employed and use a Schedule C to file. John will have to pay self-employment tax. He must file a Schedule SE if net earnings are $400 or more. Tough question / answer.


Related study sets

BIO 181 Exam 4 Homework Questions

View Set

Com 202 midterm/ final after 143

View Set

TEAS - English - Knowledge of Language

View Set

Life - Policy Provisions, Options And Riders - Terms to know

View Set

English 316.50A/50B Quiz Questions

View Set

Chapter 57: Drug Therapy for ADHD and Narcolepsy

View Set